discuss hardest assumption question.

This topic has expert replies
Legendary Member
Posts: 1404
Joined: Tue May 20, 2008 6:55 pm
Thanked: 18 times
Followed by:2 members

discuss hardest assumption question.

by tanviet » Sat Sep 05, 2009 4:22 am
this is question 27, test 4, from 15 test sets, a good sourse

in an attempt to reduce the crime rate, the governor is getting tough on criminals and making prison conditions harsher.Part of this effort has been to deny inmates the access they formerly had to college level course. However, this action is clearly counter to the governor's ultimate goal, since after being released from prison, inmates who had taken such courses commited far fewer crimes overall than other inmates

which of the following is an assumption on which the argument depend

a, not being able to take college level course while in prison is unlikely to deter anyone from a crime that he or she might otherwise have commited

b,Former inmates are no more likely to commit crimes than are members of the general po;ulation.

c,the group of inmates who chose to take college level courses were not already less likely than other inmates to commit crimes after being released

d, Taking high school level course in prison has less effect on an inmate's subsequent behavior than taking college level courses does

e,the governor
s ultimate goal actually is to gain popularity by convincing people that something effective is being done about crime.

it is clear that C is correct

Because conclution is that X cause Y, A said that not X then not Y

Is A a strenthening choice?. A strengthens conclusion and C is assumption of conclusion. IS THAT RIGHT? PLS HELP

Legendary Member
Posts: 876
Joined: Thu Apr 10, 2008 8:14 am
Thanked: 13 times

by ketkoag » Sat Sep 05, 2009 8:20 am
i think A weakens the argument coz the argument says that the course has an effect on the inmates and thus inmates who take the course are less likely to commit crime..
A says that course doesn't have any effect on the inmates..

Senior | Next Rank: 100 Posts
Posts: 76
Joined: Fri Dec 26, 2008 8:18 am
Thanked: 3 times

by srivas » Sat Sep 05, 2009 8:25 am
I am not convinced with option C

The argument says "the decision of the Governer not to allow prisioners to study will work counter to achieve his goal"

option A is convoluted and says unlikely to deter- unable to stop from commiting crime, not being able to take - ubale to take - double negative is positive
Gmat710,, Hyd

Master | Next Rank: 500 Posts
Posts: 175
Joined: Mon Feb 09, 2009 3:57 pm
Thanked: 4 times

by tom4lax » Sat Sep 05, 2009 5:36 pm
I would go with C, whats OA?

Master | Next Rank: 500 Posts
Posts: 159
Joined: Thu Aug 27, 2009 10:30 am
Thanked: 19 times

by bharathh » Sat Sep 05, 2009 6:18 pm
I would go with E.. C has already been stated in the argument (the last sentence).

Master | Next Rank: 500 Posts
Posts: 159
Joined: Thu Aug 27, 2009 10:30 am
Thanked: 19 times

by bharathh » Sat Sep 05, 2009 6:20 pm
I was wrong :(

This has already been posted and discussed here

https://www.beatthegmat.com/governor-and ... 14221.html

Legendary Member
Posts: 869
Joined: Wed Aug 26, 2009 3:49 pm
Location: California
Thanked: 13 times
Followed by:3 members

by heshamelaziry » Sat Sep 05, 2009 11:31 pm
GMAC are the best in manipulating the language in answer choices to make it confusing to anyone on the planet, and they expect us to solve this one in 2 minutes ! :x

If you look at C, stare at the part that says "were not less likely" it means that if they don't take the courses they are equally or more likely like the general public to commit crimes.
This supports the writer's conclusion, but the question ask to support the governor conclusion.

An assumption is a thought that is added to the premises and sometimes the conclusion refutes, not necessarily support.


Choice A

Legendary Member
Posts: 1404
Joined: Tue May 20, 2008 6:55 pm
Thanked: 18 times
Followed by:2 members

by tanviet » Sat Sep 12, 2009 12:35 am
the OA is C because evidence must be representative.

what I want you to discuss is why A is wrong

Legendary Member
Posts: 869
Joined: Wed Aug 26, 2009 3:49 pm
Location: California
Thanked: 13 times
Followed by:3 members

by heshamelaziry » Sat Sep 12, 2009 12:59 am
duongthang wrote:the OA is C because evidence must be representative.

what I want you to discuss is why A is wrong
what I want you to discuss is why A is wrong[/quote]

In an attempt to simplify the answer choice,I paraphrased this to be " not taking college classes, will not make them less likely to commit crime" In other words: no classes----->more likely to commit crimes. This is not present in neither chief statement nor in the writer's conclusion.

Let me know if this helps.

Legendary Member
Posts: 1404
Joined: Tue May 20, 2008 6:55 pm
Thanked: 18 times
Followed by:2 members

by tanviet » Tue Sep 15, 2009 7:37 pm
is that right that

A support conclution and answer STREGHTHEN QUESTION

C answer ASSUMPTION QUESTION.

pls, help

Legendary Member
Posts: 869
Joined: Wed Aug 26, 2009 3:49 pm
Location: California
Thanked: 13 times
Followed by:3 members

by heshamelaziry » Tue Sep 15, 2009 8:37 pm
bharathh link above says OA is A. Here you say OA is C. What the hell is this ? :evil:

Master | Next Rank: 500 Posts
Posts: 300
Joined: Sat Aug 22, 2015 10:33 am

by kris77 » Sun May 15, 2016 4:22 pm
Answer A seems to be logical one out of other answer choices